Part A: A landscape service charges costumers a one-time fee and an hourly rate of $15. For 3 hours of work m, it charges $75 Write the equation in point-slope formHow much does the landscape service charge for 20 hours

Answers

Answer 1

The hourly rate is the slope m, and we're given the point (3,75)

Now, using the point-slope form:

[tex]y-75=15(x-3)[/tex]

Now, let's put it in the slope-intercept form (clear y):

[tex]\begin{gathered} y-75=15x-45 \\ \rightarrow y=15x+30 \end{gathered}[/tex]

For 20 hours of service,

[tex]\begin{gathered} y=15(20)+30 \\ \rightarrow y=330 \end{gathered}[/tex]

The service would charge $330


Related Questions

And each diagram below, right, the 2 number on the sides of the acts that are multiplied together to get the top number of the x, but added together to get the bottom number of the x.

Answers

-34 = x*y

15 = x +y

17 and -2

__________________________

_____________

9 = x*y

6 = x +y

_

There are 38 coins in a collection of 20 paise coins and 25 paise coins. If the total value of the collection is Rupees 8.50, how many of each are there?

Answers

We will have the following:

*First:

**We stablish that x will represent the number of 20 paise coins.

**We stablish that y will represent the number of 25 paise coins.

Second: From this we will then have:

[tex]x+y=38[/tex]

&

[tex]20x+25y=850[/tex]

[This 850 is due to the fact that 8.50 Rupees are equal to 850 paise].

*Third: We solve for either x or y in the first equation:

[tex]x=38-y[/tex]

Now, we replace this in the second equation and solve for y:

[tex]20(38-y)+25y=850\Rightarrow760-20y+25y=850[/tex][tex]\Rightarrow5y=90\Rightarrow y=18[/tex]

So, we have that there are 18 25 paise coins.

Now, using this we solve for x in the first equation:

[tex]x+18=38\Rightarrow x=20[/tex]

So, we have that there are 20 20 pais coins.

How is the series 7 + 13 + 19+...+ 139 represented in summation notation?

Answers

Each term is 6 greater than the previous term.

First term is "7".

So,

a = 7

d = 6

Let's find the formula for the series,

[tex]\begin{gathered} a+(n-1)d \\ 7+(n-1)(6) \\ 7+6n-6 \\ 6n+1 \end{gathered}[/tex]

We can immediately eliminate the firsst and third choice.

The variable is "t", so the general formula will be:

[tex]6t+1[/tex]

How many terms are there?

The series starts from t = 1,

since 6(1) + 1 = 6 + 1 = 7

and 6(2) + 1 = 12 + 1 = 13

The terms match!

So, 2nd answer choice is correct!!

Answer[tex]\sum ^{23}_{t\mathop=1}(6t+1)[/tex]

Consider a die rolling game that costs $15 per play. Rolling a 6 wins you $40, rolling a 5 gets you$25, and rolling anything else gets you nothing. Find the expected value and determine if the gameis fair.

Answers

Explanation

We need to present the information given in a table as follows:

The second column shows the probability of obtaining each event. Since they are equiprobable events, their probability are 1/6.

The next column determine the cost of obtaining each event; keeping in mid that each play costs $15.

The last column shows the expected value.

We know that If the value of the game is zero i.e. there is no loss or gain for any player, then the game is fair.

According to the table, there is a loss of 25/6, hence, the game is not fair.

A) 9- (-22) - 13B) 9 - (-22) = -13A football team gained 9yards on one play andthen lost 22 yards on thenext. Write a sum ofintegers to find theoverall change in fieldposition-9- (-22) = -13D) None of the above

Answers

The sum of integers should be written like:

9 + (-22) = -13

because the are asking in fact for a SUM of integers

The two integers are 9 and -22

when added you get:

9 + (-22) = 9 - 22 = -13

ANYONE HELP ME WITH THE AREA OF THE FLOOR PLAN FOR THE OFFICE
NEED CLEAR EXPLAINATION AND ANSWER.

Answers

The area of the floor plan for the office is 2200 m².

We are given a diagram. The diagram shows the floor plan of an office. The height of the office is 50 meters. The length of the floor of the office is 55 meters. The width of the floor of the office is 40 meters. We need to find the area of the floor of the office.

The shape of the floor of the office is a rectangle. The area of a rectangle is calculated as the product of its length and width. Let the area of the floor of the office be represented by the variable "A".

A = 55*40

A = 2200

Hence, the area of the floor of the office is 2200 m².

To learn more about area, visit :

https://brainly.com/question/27683633

#SPJ1

8. Select the equation that has no real solution.12x + 15 = 12x - 157x +21= 21-5x-25 = 5x + 2512x+ 15 = 3(4x+5)

Answers

We will have that the equation with no real solution is:

[tex]12x+15=12x-15[/tex]

Because:

[tex]12x-12x=-15-15\Rightarrow0\ne-30[/tex]

What is the surface area of a right square pyramid with height of 3 centimeters and a base that measures 8 centimeters by 8 centimeters?

Answers

Okay, here we have this:

Considering the provided measures, we are going to calculate the requested surface area, so we obtain the following:

So to calculate the surface area we will use the following formula:

A = a(a + √(a^2 + 4h^2))

A = 8 cm(8cm + √((8 cm)^2 + 4(3 cm)^2))

A = 8 cm(8cm + √(64 cm^2 + 4*9 cm^2))

A = 8 cm(8cm + √(64 cm^2 + 36 cm^2))

A = 8 cm(8cm + √(100 cm^2))

A = 8 cm(8cm + 10 cm)

A = 8 cm(18 cm)

total surface areaA = 144 m^2

Finally we obtain that the total surface area is equal to 144 m^2

Find the exact value of s in the given interval that has the given circular function value.

Answers

Recall that:

[tex]\tan x=\frac{\sin x}{\cos x}\text{.}[/tex]

Therefore:

[tex]\tan s=1\Leftrightarrow\frac{\sin s}{\cos s}=1.[/tex]

Then:

[tex]\sin s=\cos s\text{.}[/tex]

Now, notice that:

[tex]\sin s-\cos s=-\sqrt{2}\cos (s+\frac{\pi}{4}).[/tex]

Then:

[tex]-\sqrt[]{2}\cos (s+\frac{\pi}{4})=0.[/tex]

Therefore:

[tex]\cos (s+\frac{\pi}{4})=0.[/tex]

Then:

[tex]\begin{gathered} s+\frac{\pi}{4}=\frac{\pi}{2}+n\pi, \\ s+\frac{\pi}{4}=\frac{3\pi}{2}+n\pi\text{.} \end{gathered}[/tex]

Therefore:

[tex]\begin{gathered} s=\frac{\pi}{4}+n\pi, \\ s=\frac{5\pi}{4}+n\pi\text{.} \end{gathered}[/tex]

Since:

[tex]s\in\lbrack\pi,\frac{3\pi}{2}\rbrack^{},[/tex]

we get that:

[tex]s=\frac{5\pi}{4}\text{.}[/tex]

Answer:

[tex]s=\frac{5\pi}{4}\text{.}[/tex]

Latoya cut a circle into & equal sections and arranged the pieces to form a shape resembling a parallelogram. So in of

Answers

Based on the diagram, the base length of the new shape is half the circumference of the circle as indicated by 1/2C.

Which of the following equations does the graph below represent?

A. -3x - 6y = 36

B. -3x + 6y = 36

C. x + 6y = 36

D. 3x + 6y = 36

Answers

The equation of the line which represents the given graph is -3x + 6y = 36

We can infer from the graph that the equation of the line passes through the points (0,6) and (-12,0).

Therefore the slope of the line passing through these two points is given by:

m = (0-6)/(-12-0)

or, m = 1 / 2

Now we can use the general equation of the line to get the required equation.

y-6 = 1/2 (x-0)

or, 2y - x = 12

Now we will multiply throughout by 3 we get:

6y - 3x = 36

or, -3x + 6y = 36

The general equation for a straight line is y = mx + c, where m stands for the line's slope and c for its y-intercept. The most frequently used straight line equation is one that has to do with geometry.

There are numerous ways to express the equation of a straight line, such as slope-intercept form,  point-slope form, standard form, general form,  etc.

Hence the required equation is -3x + 6y = 36 .

To learn more about equation visit:

https://brainly.com/question/2564656

#SPJ1

Greetings, i need help with this math problem. Thank you

Answers

The numerator of the left hand side can be rewritten as:

[tex]x^2+6x+9=(x+3)^2[/tex]

Then, the given equation can be written as:

[tex]\frac{(x+3)^2}{x+3}=0[/tex]

Since

[tex](x+3)^2=(x+3)(x+3)[/tex]

we have

[tex]\frac{(x+3)(x+3)}{x+3}=0[/tex]

We can to can cancel out one term x+3 and get

[tex]x+3=0[/tex]

which gives

[tex]x=-3[/tex]

Finally, in order to check that this value corresponds to a real answer, we need to subsitute this value into the equation and compute the limit when x approaches to -3, that is,

[tex]lim_{x\rightarrow-3}\frac{x^2+6x+9}{x+3}[/tex]

which gives

[tex]l\imaginaryI m_{x\operatorname{\rightarrow}-3}\frac{x^{2}+6x+9}{x+3}=\frac{0}{0}[/tex]

Since the limit has the form 0/0 we can to apply L'Hopital rule, that is,

[tex]l\imaginaryI m_{x\operatorname{\rightarrow}-3}\frac{\frac{d}{dx}(x^2+6x+9)}{\frac{d}{dx}(x+3)}[/tex]

which gives

[tex]l\imaginaryI m_{x\operatorname{\rightarrow}-3}\frac{\frac{d}{dx}(x^{2}+6x+9)}{\frac{d}{dx}(x+3)}=l\imaginaryI m_{x\operatorname{\rightarrow}-3}\frac{2x+6}{1}=\frac{0}{1}=0[/tex]

Since the limit exists and is equal to zero then the solution of the equation is: x= -3

The park near Amber's house has a path around its perimeter 3 that is mile long. Amber's goal is to walk 4.5 miles a day. If 4 Amber reaches her daily goal, how many times will Amber walk around the park?

Answers

We know that

• The path is 3 miles long.

,

• Amber's goal is 4.5 miles a day.

To find the number of times she will walk around the park, we have to divide.

[tex]\frac{4.5}{3}=1.5[/tex]Hence, Amber will walk around 1 entire lap and a half.

i don't know how to identify the domain and range of the graph

Answers

The domain is the set of all possible values for x. All x values ( horizontal axis) that are going to be used

Domain = (-4,-1,0,4)

The range is the set of all possible y-values . All the y-values (vertical axis) that are used.

Range = (-5)

please help me Solve for a6=a/4+2-6+x/4=-59x-7=-70=4+n/5-4=r/20-52(n+5)=-2-9x+1=-80144=-12(x+5)10-6v=-104

Answers

6=a/4+2

Subtract 2 from both sides of the equation:

6-2 = a/4+2-2

4 = a/4

Multiply both sides by 4

4 (4) = a/4 (4)

16 = a

a= 16

We use absolute value to find distances in the real world. Suppose you travel 10 miles north to the grocerystore, then 6 miles south to the post office. From there, you travel 8 miles north to the nearest bank. What isthe total distance you have traveled?A. 4 milesB. 8 milesC.12 milesD. 24 milesPlease select the best answer from the choices providedOAOBOCOD

Answers

Given:

Thesis about question is given

Required:

To select which option is correct

Explanation:

assumption S1 is the distance to the grocery store

assumption S2 is the distance to the south post office

assumption S3 is the distance to the north bank

assumption S is the total distance

according to the question

S1= 10 miles S2=6 miles S3=8 miles

S=S1+S2+S3=10+6+8=24 miles

Required answer:

option D

The total number of photos on Hannah's camera is a linear function of how long she was in Rome. She already had 44 photos on her camera when she arrived in Rome. Then she took 24 photos each day for 6 days. What is the initial value of the linear function that represents this situation? 24 photos 44 photos 6 days o days per day

Answers

she already had 44 photos

She took 24 photos each day for 6 days.

The function is:

y= 44+24x

where y is the total number of photos and x is the number of days.

The initial value is when x=0

y= 44+24(0) =44

The initial value is 44.

A survey asked "Do you think the president is doing a great job?" Of the 1200 Americans surveyed, 800 responded yes. For a 95% level of confidence find the sample proportion and the margin of error associated with the poll.

Answers

We have here working with estimating a population proportion.

We have the following information from the question:

• The sample size, n, is equal to 1200 (n = 1200).

,

• We have that the fraction that responded "Yes" is 800.

,

• We need to find a 95% level of confidence for the margin error associated with the poll.

Now, we have the sample proportion for the sample size, n = 1200 is as follows:

Sample Proportion[tex]\begin{gathered} \hat{p}=\frac{800}{1200}=\frac{2}{3}\approx0.666666666667\approx0.67 \\ \\ \hat{p}=\frac{800}{1200}\approx0.67 \end{gathered}[/tex]

Therefore, the sample proportion for the sample size is 800/1200, which is, approximately, 0.67.

The margin of error associated with the poll

The margin of error, in this case, is given by the next formula:

[tex]E=Z_c\sqrt{\frac{\hat{p}(1-\hat{p})}{n}}[/tex]

Where:

[tex]\begin{gathered} Z_c\text{ is the critical value for a 95\% level of confidence} \\ \\ n\text{ is the sample size \lparen n = 1200\rparen} \\ \\ \hat{p}\text{ is the sample proportion \lparen800/1200\rparen} \\ \end{gathered}[/tex]

Now, we have that, for a level of confidence of 95%, the critical value is equal to z = 1.96:

Now, using all of the values at our disposal, we can use the formula to find the margin of error as follows:

[tex]\begin{gathered} E=Z_c\sqrt{\frac{\hat{p}(1-\hat{p})}{n}} \\ \\ E=1.96\sqrt{\frac{\frac{8}{12}(1-\frac{8}{12})}{1200}} \\ \\ E=0.0266722216436\approx0.027 \end{gathered}[/tex][tex]\begin{gathered} E=Z_c\sqrt{\frac{\hat{p}(1-\hat{p})}{n}} \\ \\ E=1.96\sqrt{\frac{\frac{8}{12}(1-\frac{8}{12})}{1200}} \\ \\ E=0.0266722216436\approx0.027 \end{gathered}[/tex]

Therefore, in summary, we have that:

1. The sample proportion is:

[tex]\hat{p}=\frac{800}{1,200}\approx0.67[/tex]

2. The margin of error associated with the poll is:

[tex]E=0.0266722216436\approx0.027[/tex]

Find the area of a rectangle that is 3 3 over 4 inches long by 2 1 fourth inches wide. ANS.( Use mixed number) _______. in squared

Answers

Let's begin by listing out the given information:

[tex]\begin{gathered} Length(l)=3\frac{3}{4} \\ Width(w)=2\frac{1}{4} \\ Area=l\cdot w \\ Area=3\frac{3}{4}\cdot2\frac{1}{4} \\ Area=\frac{15}{4}\cdot\frac{9}{4}=\frac{15\cdot9}{4\cdot4} \\ Area=\frac{135}{16}=8\frac{7}{16} \\ Area=8\frac{7}{16}in^2 \\ \\ \therefore Area=8\frac{7}{16}in^2 \end{gathered}[/tex]

a rectangle with a base of 6 and height of 4 has been scaled with a scale factor of 3. what is the area of the scaled copy?

Answers

We have a rectangle of base 6 and height 4. We have a rectangle like this:

Now, we have a scale factor of 3. We need to multiply each side by 3:

Now the rectangle has a height of 12 and a base of 18.

The area of a rectangle is h*b. Then, the area of the scaled copy is:

[tex]A=12\cdot18\Rightarrow A=216[/tex]

Therefore, the scaled copy has an area of 216 square units.

Hi can someone please give a full explanation on how to solve this problem? I’ll give Brainliest

Answers

Answer:

C is correct.

Step-by-step explanation:

The area of the rectangle is 10 × 6, or 60 square meters.

The area of the triangle is 1/2 × 7 × 6, or 21 square meters.

So the area of the shaded region is the area of the rectangle minus the area of the triangle. That area is 60 - 21, or 39 square meters. C is correct.

make an equation to find the area of rectangle. move number and symbols to the line

Answers

Area of a rectangle

We know that the multiplication of both sides of the rectangle is the area of it.

In this case

Area: 6 x 8 = 48

A multiple choice test contains 10 questions with 5 answer choices. What is the probability of correctly answering 5 questions if you guess randomly on each question?A. 0.9936B. 0.2C. 0.0264D. 0.0003

Answers

If there are 10 questions with 5 answer choices, then first we need to find out the probability of getting the first questions randomly correct.

Therefore, that is:

[tex]\begin{gathered} Probability\text{ of getting 1 answer correct= }\frac{1}{5} \\ \\ Then\text{ if we need to get the second question correct it is:} \\ \frac{1}{5}(first\text{ question\rparen x }\frac{1}{5}=\text{ \lparen}\frac{1}{5})^2 \\ \\ And\text{ for the other questions applies the same. Therefore, if we need 5 correct answers, then:} \\ \frac{1}{5}\text{ x }\frac{1}{5}x\frac{1}{5\frac{}{}}x\frac{1}{5}x\text{ }\frac{1}{5}=\text{ \lparen}\frac{1}{5})^5\text{ = }\frac{1}{3125}=\text{ 0.0003} \end{gathered}[/tex]

The answer is D. 0.0003

2705 is compound annually at a rate of 8% for 1 year

Answers

The formula of compound interest is,

[tex]\begin{gathered} A=P(1+i)^n \\ \text{Here, A=2075, i=8\%, n=1 year} \\ 2075=P(1+\frac{8}{100})^1 \\ P=1921.3 \end{gathered}[/tex]

A farmer has 36 ft of fencing and wants to enclose the maximum rectangular area for his llamas. Find the dimensions of three possible areas he could enclose. What do you think the maximum area is? Why?

Answers

The farmer would like to eclose a rectangle

First we know that the perimeter of the rectangle is

[tex]P=2x+2y[/tex]

We also know that we have 36 ft of fence, that is we only can enclose a rectangle of 36 ft of perimeter. Then

[tex]36=2x+2y[/tex]

From this we can find y

[tex]\begin{gathered} 36=2x+2y \\ 36-2x=2y \\ y=\frac{36-2x}{2} \\ y=18-x \end{gathered}[/tex]

The area of the rectangle is

[tex]A=xy[/tex]

But we know the value of y, plugging this value into the last equation we have that

[tex]A=x(18-x)[/tex]

To find three possible values for the area we only have to give values to x. This values have to be positive (since we can't have a negative lenght). We also notice that the value can't exceed 18 since that would mean a zero area. With those point in consideration we choose three values between zero and 18.

If x=3, then the area is

[tex]\begin{gathered} A=3(18-3) \\ =3(15) \\ =45 \end{gathered}[/tex]

If x=9, then the area is

[tex]\begin{gathered} A=9(18-9) \\ =9(9) \\ =81 \end{gathered}[/tex]

if x=15, then the area is

[tex]\begin{gathered} A=15(18-15) \\ =15(3) \\ =45 \end{gathered}[/tex]

Then we have three possible areas for the rectangle.

The maximum value for the area is 81, and we see that because the equation for the area is a parabola that opens down with vertex in the point (9,81)

Find the area of quadrilateral ABCD. Round the area to the nearest whole number, if necessary.у| A(-5,4)4B(0, 3)22F(-2,1)-226 xTC(4, -1)-4E(2, -3)D(4, -5)6The area issquare units.

Answers

We have a quadrilateral ABCD and we want to calculate the area.

We can divide it in three areas (two triangles and one rectangle) and then add the surfaces.

As it is rotated 45 degrees, we can define a "new unit" that is the diagonal of a square of 1 by 1 unit, in the scale of the graph.

This new unit, the diagonal that we will call "d", by the Pythagorean theorem, has a value of:

[tex]d=\sqrt{2}[/tex]

We will start then with the triangle ABF. It has a side BF that has a value of 2 diagonals (2d) and a side FA that has a value of 3 diagonals (3d). The area of a triangle is half the multiplication of this two sides, so we have:

[tex]\frac{\bar{BF}\cdot\bar{FA}}{2}=\frac{2d\cdot3d}{2}=3d^2=3(\sqrt{2})^2=3\cdot2=6[/tex]

The second triangle is CED. We repeat the process and we have:

[tex]\frac{\bar{CE}\cdot\bar{ED}}{2}=\frac{2d\cdot2d}{2}=2d^2=2\cdot2=4[/tex]

The rectangle BCEF has an area of:

[tex]\bar{BF}\cdot\bar{EF}=2d\cdot4d=8d^2=8\cdot2=16[/tex]

Now we have the three areas. If we add them we get the area of ABCD:

[tex]6+4+16=26[/tex]

The quadrilateral ABCD has an area of 26 units^2.

taxes :2.69 * 10 ^ 5 square miles Rhode Island:1.21 * 10 ^ 3 square miles Determine the differences in square miles between the area Texas and the area Rhode Island. write your answer in scientific notation

Answers

Explanation:

The difference is:

[tex]2.69\cdot10^5-1.21\cdot10^3[/tex]

To solve this we need the exponent of 10 be the same for both terms of the substraction. It is better is we change the exponent of the area of Texas, by moving the decimal point two places to the right:

[tex]2.69\cdot10^5-1.21\cdot10^3=269\cdot10^3-1.21\cdot10^3[/tex]

Now we can substract the numbers and leave the 10³ out:

[tex]269\cdot10^3-1.21\cdot10^3=(269-1.21)\cdot10^3=267.79\cdot10^3[/tex]

Scientific notation has always only one place with a number before the decimal point. Therefore 267.79x10³ is not in scientific notation, we have to move the decimal point two places to the left and add 2 to the exponent of 10:

Answer:

The difference is 2.6779 x 10⁵ square miles

graph a piecewise function with 3 equations and sketch a graph

Answers

Solution:

Given:

[tex]h(x)=\begin{cases}2x,x\le-2 \\ x^2-1,-2A piecewise function is a function that is defined by different formulas or functions for each given interval.

It is a function in which more than one formula is used to define the output over different pieces of the domain.

The function h(x) given has three outputs for three different domains.

[tex]\begin{gathered} \text{The first is a linear function;} \\ h(x)=2x \\ \\ \text{The second is a quadratic function;} \\ h(x)=x^2-1 \\ \\ \text{The third is a linear function;} \\ h(x)=x-3 \end{gathered}[/tex]

Therefore, the graph using a graph plotter (desmos) is as shown below;

If f(x) = 2x^3 + 10x^2 + 18x + 10 and x + 1 is a factor of f(x), then find all of the zeros of f(x) algebraically

Answers

Given the polynomial:

[tex]f(x)=2x^3+10x^2+18x+10[/tex]

We know that (x + 1) is a factor of f(x). We divide f(x) by (x + 1):

Then:

[tex]f(x)=(x+1)(2x^2+8x+10)=2(x+1)(x^2+4x+5)[/tex]

For the quadratic term, we solve the following equation:

[tex]x^2+4x+5=0[/tex]

Using the general solution for quadratic equations:

[tex]\begin{gathered} x=\frac{-4\pm\sqrt{4^2-4\cdot1\cdot5}}{2\cdot1}=\frac{-4\pm\sqrt{16-20}}{2}=\frac{-4\pm\sqrt{4}}{2} \\ \\ \therefore x=-2\pm i \end{gathered}[/tex]

The zeros of f(x) are:

[tex]\begin{gathered} x_1=-1 \\ \\ x_2=-2-i \\ \\ x_3=-2+i \end{gathered}[/tex]

Question 4 of 10Select the correct product of the exponential expression.35A. 5.5.5B. 3.5C. 15D. 3.3.3.3.3SUBMIT

Answers

You have the following expression:

[tex]3^5[/tex]

it means that the number 3 is multipled by itself five times (because of the exponent), then, the previous expression can be written as follow:

[tex]3^5=3\cdot3\cdot3\cdot3\cdot3[/tex]

Other Questions
What is the positive solution for x^3 + 3x - 9 = x -1 + 2x? Elizabeth wraps a gift box in the shape of a square pyramid. The figure below shows a net for the gift box. 6 in 6.8 in what can you prove congruent from your given what would happen to the distances between bright spots (intensity maxima) on the screen if the spacing between the slits is doubled? group of answer choices The aspect ratio for a computer screenis 118. If the width of the computer screen is 13.5inches. What is the begi of the mean?Answer question 1 photo attached What about takes four hours to travel 128 km going upstream and return it takes two hours going down stream what is the rate of the boat in Stillwater and what is the rate of a Current I need help with one of my physics question The equation that models Earth's elliptical orbit around the sun is (x+2.5)^2/22,350.25+y^2/22,344=1 in millions of kilometers. If the sun is located at one focus and its coordinates are (0,0), find Earth's farthest distance from the sun in millions of kilometers. If a 150 pound weight is on a frictionless surface, raised at an angle of 35 degrees, what is the tension in the rope that keeps it from sliding down? What is the force perpendicular to the surface? What constant torque is required to bring it up to an angular speed of 400 rev/min in 8. 00 s , starting from rest?. octavia simplified the expression as shown explain octavias error and correct her work There are 83 skiers in line to ride the chair lift. If each chair seats 6 people, how many skiers will be on the last chair? I need help getting the answer on all of theseFirst one As the object travels along the track, what is the maximum height that it reaches above point E? What are the OSHA PEL-TWyA and PEL-STEL values for nitric acid? 2. Damian is buying movie tickets to a movie. The tickets cost $4.35 per ticket. Damian has $40.00. What is the greatest amount of tickets he can buy? Logarithmic help is needed. Be sure to note the differences between logarithmic and exponential forms in each equation. (n-1)9. Expand11-11 + 2 + 3 + 4 + 5 + 60 + 1 + 2 + 3 + 4 + 5 + 60 + 1 + 2 + 3 +4 + 5AB(-1) + (-2) + (-3) + (-4) + (-5) + (-6) you are tasked with helping plan a school dance. you expect 110 people to attend the dance and need to provide seating for each person. the tables come in two different sizes. the smaller table can seat 2 people, and the larger tables can seat 4 people.Write an equationgraph the line for the equation you wroteYALL PLEASEEEEEEEEEEEEEEEE I NEED YALLS HELP write as a product:y raised to the 3 - y raised to the 5